Đến nội dung

xuanhoan23112002 nội dung

Có 95 mục bởi xuanhoan23112002 (Tìm giới hạn từ 30-03-2020)



Sắp theo                Sắp xếp  

#709806 Tìm GTLN của $2x^2-3xy-2y^2$

Đã gửi bởi xuanhoan23112002 on 03-06-2018 - 07:30 trong Bất đẳng thức và cực trị

Ta có:

$A-36a=(2-25a)x^2-(3+20a)xy-(2-40a)y^2$

Coi phương trình trên là phương trình bậc 2 ẩn x tìm giá trị của a sao cho phương trình có nghiệm kép tức là$\Delta =0$ (Chú ý: Tìm giá trị lớn nhất thì $A-36a$ mang dấu trừ của 1 bình phương đủ nên $2-25a<0, 2-40a<0$)

Từ đó tìm được: $a=\frac{1}{12 }$




#709756 Tìm GTLN của $2x^2-3xy-2y^2$

Đã gửi bởi xuanhoan23112002 on 02-06-2018 - 07:23 trong Bất đẳng thức và cực trị

Đặt $A=2x^2-3xy-2y^2$

$\Leftrightarrow$$A-3=2x^2-3xy-2y^2-\frac{1}{12}(25x^2-20xy+40y^2)$

$\Leftrightarrow$$A-3=-\frac{1}{12}x^2-\frac{4}{3}xy-\frac{16}{3}y^2$

$\Leftrightarrow$$A-3=-\frac{1}{12}(x+8y)^2\leq 0$

$\Leftrightarrow A\leq 3$

Đẳng thức xảy ra $\Leftrightarrow (x, y)=(\frac{4\sqrt{2}}{5}, -\frac{\sqrt{2}}{10})$ hoặc $(x, y)=(-\frac{4\sqrt{2}}{5}, \frac{\sqrt{2}}{10})$

Vậy Max của $2x^2-3xy-2y^2=3$




#707185 Tìm giá trị nhỏ nhất của: $S=a_{1}^{2}+a_{2...

Đã gửi bởi xuanhoan23112002 on 28-04-2018 - 15:19 trong Bất đẳng thức - Cực trị

Problem: Cho $a_{1}, a_{2},...,a_{19}$ là các số tự nhiên thỏa mãn: $a_{1}+a_{2}+...+a_{19}=26.$ Tìm giá trị nhỏ nhất của: $S=a_{1}^{2}+a_{2}^{2}+...+a_{19}^{2}.$

 




#704997 tìm giá trị của x để a nguyên

Đã gửi bởi xuanhoan23112002 on 05-04-2018 - 21:04 trong Đại số

PP làm: lấy tử chia mẫu được phần nguyên và phân số rồi sử dụng tính chia hết




#705205 Tìm a? đề hàm số có đạo hàm tại x=-1

Đã gửi bởi xuanhoan23112002 on 08-04-2018 - 08:32 trong Hàm số - Đạo hàm

Ta phải xét tính liên tục của hàm số và đặt điều kiện đạo hàm trái = đạo hàm phải bạn nhé




#710187 Tâm đường tròn ngoại tiếp tứ giác ABCD nằm trên SI

Đã gửi bởi xuanhoan23112002 on 07-06-2018 - 11:51 trong Hình học

Cho tứ giác ABCD ngoại tiếp đường tròn tâm I. Giả sử bên trong tứ giác ta vẽ được 4 đường tròn bằng nhau và cùng đi qua 1 điểm S, và mỗi đường tròn tiếp xúc với 2 cạnh liên tiếp của tứ giác đó. Chứng minh tứ giác ABCD nội tiếp 1 đường tròn và tâm đường tròn đó nằm trên SI.




#705203 Trong một giải đấu bóng đá có 10 đội tham gia theo thể thức mỗi đội đều gặp đ...

Đã gửi bởi xuanhoan23112002 on 08-04-2018 - 08:27 trong Tổ hợp và rời rạc

Ta chứng minh bài toán bằng phản chứng( cả 1 và 2 đều không xảy ra)

Gọi 10 đội bóng là a(i là số tự nhiên và i chạy từ 1 đến 10)

Giả sử a10 là đội bóng có số trận thua nhiều nhất

Khi đó nếu tồn tại giá trị i từ 1 đến 9 mà a10 thang ai thì tất cả cả đội bóng mà a10 thua thì ai cũng thua (vô lí do a10 có số trận thua nhiều nhất)

Suy ra a10 thi đấu với các đội còn lại chỉ có thể hòa hoặc thua

Mà theo gia sư điều kiện 2 không xảy ra nên a10 thua ít nhất 7 đội là aj (j chạy từ 1 đến 7)

Lập luận tương tự như trên với a7 là đội có số trận thua nhiều nhất trong 7 đội trên thì a7 phải thừa ít nhất 4 đội giả sử là: a1, a2, a3, a4.

Lập luận tương tự như trên với a4 là đội có số trận thua nhiều nhất trong 4 đội trên thì a4 phải thừa ít nhất 1 đội giả sử là: a1

Như vậy ta tìm được 4 đội: a1, a4, a7, a10, lập thành 4 đội thỏa mãn điều kiện 1( mâu thuẫn với giả sử)

Do đó giả sử sai. Ta có điều phải chứng minh




#669686 Trong 100.000.000 số hạng đầu tiên của dãy Fibonacci, có tồn tại hay...

Đã gửi bởi xuanhoan23112002 on 24-01-2017 - 13:57 trong Tổ hợp và rời rạc

Ký hiệu $F_{n}$ là số hạng thứ $n$ của dãy Fibonacci

Chia mỗi số $F_{n}$ cho $10^4$ đc các số dư là $r_{n}$ (mod $10^4$) với $r_{i}=\overline{0,9999}$

Xét tất cả các cặp có thể có của các số nguyên từ $0$ đến $9999$ có $10^8$ cặp $(r_{0};r_{0});(r_{0};r_{1});(r_{1};r_{2});...$

Giả sử ko có cặp nào là $(0;0)$ thì các cặp trên phải có 2 cặp trùng nhau theo nguyên lý Dirichlet, gsử:

  $r_{n}=r_{p}$ và $r_{n+1}=r_{p+1}$ ($n>p$)

$\Rightarrow r_{n-1}=r_{p-1} \Rightarrow r_{n-2}=r_{p-2} \Rightarrow ... \Rightarrow r_{1}=r_{n-p+2}=r_{n-p+1} \Rightarrow r_{n-p+1}=r_{n-p+2} \Rightarrow r_{n-p}=0$

Vậy tồn tại $1$ số $F_{n}$ chia hết cho $10^4$ hay có tận cùng là $4$ chữ số $0$

Mình không hiểu bài này lắm, gửi lại cho mình đi.




#706068 Topic ôn thi hình học vào cấp 3 chuyên

Đã gửi bởi xuanhoan23112002 on 16-04-2018 - 21:14 trong Hình học

Bài 19 (Đề thi vào lớp 10 chuyên Nam Định 2017):Cho tam giác ABC có ba góc nhọn nội tiếp đường tròn (O), AB < AC. Các tiếp tuyến của đường tròn (O) tại B và C cắt nhau tại M. Đường thẳng qua M song song với AB cắt đường tròn (O) tại D và E (D thuộc cung nhỏ BC), cắt BC tại F, cắt AC tại I.

1) Chứng minh năm điểm M, B, O, I, C cùng thuộc một đường tròn

2)Chứng minh$\frac{FI}{FE}=\frac{FD}{FM}$

3) OI cắt (O) tại P và Q (P thuộc cung nhỏ AB). QF cắt (O) tại T (T khác Q). Tính tỉ số $\frac{TQ^2+TM^2}{MQ^2}$




#704835 TOPIC thảo luận, trao đổi toán thi học sinh giỏi khối 10,11 .

Đã gửi bởi xuanhoan23112002 on 03-04-2018 - 22:54 trong Chuyên đề toán THPT

Đóng góp bài này cho topic

Cho a,b,c,d  thỏa mãn a+b+c+d=6 và a2+b2+c2+d2=12

cmr 48 >= 4(a3+b3+c3+d3) -(a4+b4+c4+d4) >= 36

Lời giải:
4(a3+b3+c3+d3) - (a4+b4+c4+d4) = -((a-1)4+(b-1)4+(c-1)4+(d-1)4)+52
Đặt x=a-1, y=b-1, z=c-1, t=d-1.
Bất đẳng thức cần chứng minh tương đương với:
 16>= x4+y4+z4+t4>=4
Ta cũng có: x2+y2+z2+t2=4
Từ đây áp dụng bất đẳng thức AM-GM ta có điều phải chứng minh(Q.E.D) 
Đẳng thức xảy ra tại (a,b,c,d) = (3,1,1,1) và (2,2,2,0) và các hoán vị của nó



#709553 số học

Đã gửi bởi xuanhoan23112002 on 30-05-2018 - 07:32 trong Số học

Từ giả thiết ta thấy ngay a, b, c đều là các số lẻ mà một số chính phương lẻ chia 8 dư 1

Từ nhận xét trên: $a^{30}+b^{4}+c^{2018}\equiv 3$ (mod 8)




#705630 Số học

Đã gửi bởi xuanhoan23112002 on 12-04-2018 - 20:09 trong Số học

Tìm các số nguyên dương n sao cho: Với mọi a, b là các số nguyên dương, nếu a2b+1 chia hết cho n thì a2+b cũng chia hết cho n




#709566 Số chính phương

Đã gửi bởi xuanhoan23112002 on 30-05-2018 - 09:39 trong Số học

Bài toán này sử dụng phương pháp bước nhảy Viete. Các bài viết khác về bước nhảy Viete trên VMF

http://diendantoanho...ước-nhảy-viete/

Lời giải của bài toán trên bạn có thể tham khảo ở đây: http://math.stackexc...-its-an-integer




#668399 MỘT SỐ PHƯƠNG PHÁP GIẢI TOÁN TỔ HỢP THCS

Đã gửi bởi xuanhoan23112002 on 15-01-2017 - 12:32 trong Toán rời rạc

ko vi no ch

 

Bài 15 : Một bạn cờ quốc tế $8\times 8$  . Hỏi rằng quân mã có thể đi nước đầu tiên từ ô dưới cùng bên trái và kết thúc ở ô trên cùng bên phải không ? Với điều kiện nó phải đi qua tất cả các ô trên bàn cờ và mỗi ô chỉ đi qua đúng một lần. 

ko vi quan ma ko the di het ban co




#669654 MỘT SỐ PHƯƠNG PHÁP GIẢI TOÁN TỔ HỢP THCS

Đã gửi bởi xuanhoan23112002 on 24-01-2017 - 06:11 trong Toán rời rạc

Trên mặt phẳng cho 5 điểm có tọa độ nguyên trong đó không có 3 điểm nào thẳng hàng .Chứng minh rằng trong số các tam giác tạo thành từ 5 điểm đã cho có ít     nhất 3 tam giác có diện tích nguyên




#704920 Lập phương trình đt d qua M

Đã gửi bởi xuanhoan23112002 on 04-04-2018 - 22:14 trong Phương pháp tọa độ trong mặt phẳng

Cho điểm M(2;1). Lập d đi qua M cắt Ox, Oy tại A, B sao cho khoảng cách từ O đến d là max

d:ax+by-2a-b=0

(a,b khác 0)

Từ đây ta tìm được tọa độ giao điểm của (d) với Ox, Oy theo a, b

Áp dụng hệ thức lượng trong tam giác vuông tính được khoảng cách từ O đến d theo a,b

Tìm max của giá trị.




#706221 Dạng toán: Trò chơi

Đã gửi bởi xuanhoan23112002 on 17-04-2018 - 21:40 trong IQ và Toán thông minh

max khó




#705303 Cần lắm một lời giải thích !

Đã gửi bởi xuanhoan23112002 on 08-04-2018 - 23:22 trong Hình học phẳng

Đó là tính chất góc định hướng được tạo bởi 4 điểm đồng viên




#709371 CMR: $\sum \frac{ab+c^{2}}{a+b}...

Đã gửi bởi xuanhoan23112002 on 27-05-2018 - 17:05 trong Bất đẳng thức và cực trị

Ta có: $\sum \frac{ab+c^2}{a+b}+\sum c= \sum \frac{(c+a)(c+b)}{a+b}\geq 2(a+b+c)$ (bất đẳng thức AM-GM)

$\Rightarrow \sum \frac{ab+c^2}{a+b}\geq a+b+c$

Đẳng thức xảy ra $\Leftrightarrow a=b=c> 0$

Vậy bất đẳng thức được chứng minh.




#705136 Chứng minh rằng với mọi số thực dương $a,b>2$ thì $2^a-1...

Đã gửi bởi xuanhoan23112002 on 06-04-2018 - 22:59 trong Số học

Bài toán sai khi a chia hết cho b 

Nếu a không chia hết cho b. Đặt a=bq+r(0<r<b)

Sử dụng phản chứng để suy ra 2r - 1 chia hết cho 2b - 1 (điều này vô lí do 0<2r - 1<2b - 1)

Từ đó ta có điều phải chứng minh.




#706210 Chứng minh MB vuông góc MN khó

Đã gửi bởi xuanhoan23112002 on 17-04-2018 - 20:40 trong Hình học

Bài này có thể sử dụng tích vô hướng của lớp 10




#705137 chứng minh

Đã gửi bởi xuanhoan23112002 on 06-04-2018 - 23:24 trong Số học

chứng minh rằng với mọi n$\epsilon$$\mathbb{Z}$ thì $n^{5}$ và n có chữ số tận cùng giống nhau 

n5 - n =n(n-1)(n+1)(n2+1)

Dễ dàng chứng minh n5-n chia hết cho 2

Với n chia hết cho 5, chia 5 dư 1 hoặc 4 thì n5-n chia hết cho 5

Với n chia 5 dư 2 hoặc 3 thì n2+1 chia hết cho 5

Như vậy ta có n5-n chia hết cho 5, n5-n chia hết cho 2 và gcd(5,2)=1

Nên n5-n chia hết cho 10

Hay n5 và n có chữ số tận cùng giống nhau




#705140 chứng minh

Đã gửi bởi xuanhoan23112002 on 06-04-2018 - 23:36 trong Số học

Lời giải nhanh nhất cho bài toán này là cách áp dụng định lý Fermat nhỏ:

x- x chia hết cho 5

Dễ dàng chứng minh x5 - x chia hết cho 2

Như vậy ta cũng có điều phải chứng minh




#662739 Chuyên đề : Làm mạnh BĐT CôSy

Đã gửi bởi xuanhoan23112002 on 22-11-2016 - 20:51 trong Bất đẳng thức và cực trị

Tìm giá trị nhỏ nhất của 

P=$\fn_jvn \frac{a}{bc}$+$\fn_jvn \frac{2b}{ca}$+$\fn_jvn \frac{5c}{ab}$

trong đó a2+b2+c2=6




#709360 Cho $x, y, z > 0$ và $x+y+z= 1$ . Chứng minh

Đã gửi bởi xuanhoan23112002 on 27-05-2018 - 15:33 trong Bất đẳng thức và cực trị

Theo bất đẳng thức Schur ta có:

$(x+y+z)^3+9xyz\geq 4(x+y+z)(xy+yz+zx)$

$\Leftrightarrow 9xyz\geq 4(xy+yz+zx)-1$

$\Leftrightarrow 5xyz+1\geq 4(xy+yz+zx-xyz)$

Theo bất đẳng thức AM-GM ta có:

$xyz\leq \frac{(x+y+z)^3}{27}= \frac{1}{27}$

$\Rightarrow xy+yz+zx-xyz\leq \frac{8}{27}$

Vậy bất đẳng thức được chứng minh.